Scipeáil chuig an bpríomhábhar
Microsoft
|
Math Solver
Réiteach
Cleachtadh
Seinn
Topaicí
Réamh-Ailgéabar
Meán
Mód
An Fachtóir Coitianta is Mó
An t-iolra is lú coitianta
Ord na nOibríochtaí
Codáin
Codáin Mheasctha
Príomh-Fhachtóiriú
Easpónant
Fréamhacha
Ailgéabar
Comhcheangail Cosúil le Téarmaí
Réitigh le haghaidh athróg
Fachtóir
Leathnaigh
Codáin a Mheas
Cothromóidí Líneacha
Cothromóidí Ceathairshruthacha
Neamhionannais
Córais Cothromóidí
Máithreáin
Triantánacht
Simpligh
Meastóireacht
Graif
Cothromóidí a Réiteach
Calcalas
Díorthaigh
Integrals
Teorainneacha
Ionchuir Ailgéabar
Ionchuir Triantánachta
Ionchuir Calcalais
Ionchuir Maitrís
Réiteach
Cleachtadh
Seinn
Topaicí
Réamh-Ailgéabar
Meán
Mód
An Fachtóir Coitianta is Mó
An t-iolra is lú coitianta
Ord na nOibríochtaí
Codáin
Codáin Mheasctha
Príomh-Fhachtóiriú
Easpónant
Fréamhacha
Ailgéabar
Comhcheangail Cosúil le Téarmaí
Réitigh le haghaidh athróg
Fachtóir
Leathnaigh
Codáin a Mheas
Cothromóidí Líneacha
Cothromóidí Ceathairshruthacha
Neamhionannais
Córais Cothromóidí
Máithreáin
Triantánacht
Simpligh
Meastóireacht
Graif
Cothromóidí a Réiteach
Calcalas
Díorthaigh
Integrals
Teorainneacha
Ionchuir Ailgéabar
Ionchuir Triantánachta
Ionchuir Calcalais
Ionchuir Maitrís
Bunúsach
ailgéabar
triantánacht
calcalas
Staitisticí
Maitrísí
Carachtair
Luacháil
0
Tráth na gCeist
Limits
5 fadhbanna cosúil le:
\lim_{ x \rightarrow 0 } 5x
Fadhbanna den chineál céanna ó Chuardach Gréasáin
Prove that for any c \neq 0 \lim_{x \rightarrow c}{h(x)} does not exist and that \lim_{x \rightarrow 0}{h(x)} does exist.
https://math.stackexchange.com/questions/334631/prove-that-for-any-c-neq-0-lim-x-rightarrow-chx-does-not-exist-and
Hint: take one sequence that contains only rationals and another one that contains only irrationals (both tending to c\ne 0). For the case of c=0, you can use e.g. that h is continuous at 0 ...
Proofs regarding Continuous functions 1
https://math.stackexchange.com/questions/526691/proofs-regarding-continuous-functions-1
The proof of part a) needs to be modified a bit. You have used the logic that if N \leq f(x) \leq M then xN \leq xf(x) \leq xM. This holds only when x \geq 0. It is better to change the argument ...
Use L'Hopital's with this problem?
https://math.stackexchange.com/questions/1419122/use-lhopitals-with-this-problem
Let \displaystyle y=\lim_{x\rightarrow 0^{+}}\left(\frac{1}{x}\right)^{\sin x}\;, Now Let x=0+h\;, Then \displaystyle y=\lim_{h\rightarrow 0}\left(\frac{1}{h}\right)^{\sin h} So \displaystyle \ln(y) = \lim_{h\rightarrow 0}\sin (h)\cdot \ln\left(\frac{1}{h}\right) = -\lim_{h\rightarrow 0}\sin h\cdot \ln(h) = -\lim_{h\rightarrow 0}\frac{\ln(h)}{\csc (h)}\left(\frac{\infty}{\infty}\right) ...
Calculate: \lim_{x \to 0 } = x \cdot \sin(\frac{1}{x})
https://math.stackexchange.com/questions/1066434/calculate-lim-x-to-0-x-cdot-sin-frac1x
Your proof is incorrect, cause you used incorrect transform, but it has already been stated. I'll describe way to solve it. \lim_{x \to 0}\frac{\sin(\frac{1}{x})}{\frac{1}{x}} \neq 1 Hint : ...
Prove that f(x) is bounded. Please check my proof.
https://math.stackexchange.com/q/1052420
Here is another approach: Let L_0 = \lim_{x \downarrow 0} f(x), L_\infty = \lim_{x \to \infty} f(x). By definition of the limit we have some \delta>0 and N>0 such that if x \in (0, \delta), ...
Complex Function limit by investigating sequences
https://math.stackexchange.com/questions/1915934/complex-function-limit-by-investigating-sequences
If a limit as z \to 0 exists, one should be able to plug in any sequence \{ z_n \} going to zero and get the same limit. Limits of sequences are generally easier to work with. So in this case if ...
Tuilleadh Míreanna
Roinn
Cóipeáil
Cóipeáladh go dtí an ghearrthaisce
Fadhbanna Comhchosúla
\lim_{ x \rightarrow 0 } 5
\lim_{ x \rightarrow 0 } 5x
\lim_{ x \rightarrow 0 } \frac{2}{x}
\lim_{ x \rightarrow 0 } \frac{1}{x^2}
Ar ais go barr